Difference between revisions of "AIME 2020(MOCK) Problems"

(Problem 3)
(Problem 4)
Line 17: Line 17:
  
 
==Problem 4==
 
==Problem 4==
Let <math>\lfloor \ x \ rfloor</math>
+
Let <math>\lfloor \ x \rfloor</math>

Revision as of 23:01, 25 June 2020

Problem 1

Let $N$ be $112123123412345... (1000 digits)$. What is the remainder when $N$ is divided by $21$?


Problem 2

Let $K$ be a set of polynomials $P(x)$ with integral coefficients such that the roots of $P(x)$ are $cos \frac{\pi}{7}$, $cos \frac{\pi}{11}$, and $cos \frac{\pi}{17}$. What is the least possible sum of the coefficients of $P(x)$?


Problem 3

How many $15$ digit base $5$ positive integers consist of exactly $2$ pairs of consecutive $0$s but no $4$ consecutive $3$s?


Problem 4

Let $\lfloor \ x \rfloor$